Can someone help me with all 3

Can Someone Help Me With All 3

Answers

Answer 1

Answer:

1) 20

2) 60

3) 10

4) 90

Step-by-step explanation:

1. first quartile is the beginning of the box

2. third quartile is the end of the box

3. lowest is the first point of the graph

4. vice versa of 3


Related Questions

please helpp!!!!!!!!​

Answers

Step-by-step explanation:

the answer is in picture

Find the following ratios. PLEASE HELP QUICK!!

Answers

Answer:

sin(A) = 5/13 ≈ 0.38

cos(B) = 5/13 ≈ 0.38

tan(A) = 5/12 ≈ 0.42

Step-by-step explanation:

If you need more explanations just say it :)

Answer:

I hope your dreams come true! Beileve in your self! and do ur best <3

Please help explanation if possible

Answers

Answer:

17.6

Step-by-step explanation:

Answer:

80 pennies ; 160 nickels

Step-by-step explanation:

Given the 2 equations

0.01p + 0.05n = 8.80 → (1)

n = 2p → (2)

Substitute n = 2p into (1)

0.01p + 0.05(2p) = 8.80

0.01p + 0.1p = 8.80

0.11p = 8.80 ( divide both sides by 0.11 )

p = 80

Substitute p = 80 into (2)

n = 2 × 80 = 160

There are 80 pennies ; 160 nickels

what is the solution to the equation below? sqrt x-7 = 5
A. 144
B. 12
C. 2
D. 4

Answers

Answer:

I think the answer is B.12

If this not correct, Sorry.

Help!!!! Please!! Thanks

Answers

never saw an indian guy asking for help in so simple question

Step-by-step explanation:

ans of 3rd   given statement is true, conclusion, mb=nb

ans of 4th    given statement is true, conclusion alt int angles of paralel lines are equal

AB=BA
∠ABM=∠ABN
MB=NB
△AMB≌△ANB (SAS)

∠CBM=∠ADM
∠BCM=∠DAM
∠BMC=∠DMA
△BCM∽△DAM (AAA)

Hope my answer helped u :)

Classify the polynomial 5x3 + 4x - 2 by degree.

Answers

Answer:

3

Step-by-step explanation:

3 would be the degree of the polynomial since it has the highest degree.

Find the largest integer not greater than the following expression :
[tex]\displaystyle \large \boldsymbol{} \frac{2150}{2005} +\frac{2150}{2006} +\frac{2150}{2007 } + ...+\frac{2150}{2020}[/tex]

Answers

Answer:

17

Step-by-step explanation:

The number of terms:

2020 - 2004 = 16

Each term is greater than 1, so the expression is greater than 16:

2150/2005 = 1 + 145/20052150/2006 = 1 + 144/2006...2150/2020 = 1 + 130/2020

It's easy to note each of the numbers getting smaller like 145/2005 > 144/2006 etc. Taking the smallest fraction.

The sum is:

S > 16 + 16*130/2020 = 16 + 2080/2020 > 16 + 1 = 17

So the largest integer not greater than S is 17.

About how many cells are in the human body?

You can assume that a cell is a sphere with radius 10-3 cm and that the density of a cell is approximately the density of water which is 1g/cm3.

Answers

Answer:

In other words: The human body consists of some 37.2 trillion cells.

Step-by-step explanation:

can somebody explain how they got 37.2 trillion cells.!!!

please helpppp

Ian has 300 counters in a bag. Paul
takes 42 of them. Derek takes 65 of
them. Anne takes 33 of them. What
fraction does Ian have left? Give your
answer in its simplest form.

Answers

42 + 65 + 33 = 140, so Ian has 160 counters left. 160/300 in its simplest form is 8/15

HELP ASAP

The graph of f(x)= |x| is chosen below. Write the equation for the stretched graph, g(x).

Answers

Answer:

y = |3x|

Step-by-step explanation:

From the table below, determine whether the data shows an exponential function. Explain why or why not. x 3 1 -1 -3 y 1 2 3 4 a. No; the domain values are at regular intervals and the range values have a common sum 1. b. No; the domain values are not at regular intervals. c. Yes; the domain values are at regular intervals and the range values have a common factor 2. d. Yes; the domain values are at regular intervals and the range values have a common sum 1. Please select the best answer from the choices provided A B C D

Answers

A………………………………………………….

Answer:

C

Step-by-step explanation:

you find the variable

find H.C.F OF 4x²y and xy²​

Answers

Answer:

xy

Step-by-step explanation:

The HCF is known as the highest common factor. To find the HCF of two values, we have to take the greatest number that fits into all of their factors.

To start, we can list each value's factors.

For 4x²y, this can also be written as 4*x²*y = 4*x*x*y. In multiplication, we can take the factors of each value that is multiplied. Therefore, we can start with the factors of 4 and then go to the factors of x² and so on. Our factor list is thus 1,2,4,x,x²,y

Similarly, for xy² = x*y*y, our factors are x, y, and y²

The common factors for each of these are x and y. Assuming all values are positive and greater than 1, the x*y will be greater than either x or y. Therefore, the highest common factor would be x*y = xy

Determine which equations have the same solution set as 2/3 -x +1/6 = 6x by recognizing properties, rather than solving. Check all that apply.

Answers

Answer:

The answer is "0.1190".

Step-by-step explanation:

Given:

[tex]\to \frac{2}{3} -x +\frac{1}{6} = 6x\\\\\to \frac{2}{3} +\frac{1}{6} = 6x+x\\\\\to \frac{4+1}{6} = 7x\\\\\to \frac{5}{6} = 7x\\\\\to x=\frac{5}{6\times 7} \\\\\to x=\frac{5}{42}\\ \\\to x= 0.1190[/tex]

Answer:

A.) 4 - 6x + 1 = 36x

B.) 5/6 - x = 6x

F.) 5 = 42x

Step-by-step explanation:

edge.

( 2 + 3 ) ^-1 x ( 2 ^-1 + 2^-1 )

Answers

Answer:

Step-by-step explanation:

[tex]\\\\\\\\\\\\\\\\\\\\\\\\\\\\\\\\\\\\\\\\\\\\\\\\\\\\\\\la\la\la\la\ddddddddddddddddddddddddddddddddcleverdddddd\ffffffffffffffffffffffffffffffffffffffff\pppppppppppppppppppppppppppppppppppp\ddddddddddddddddddd\displaystyle\ \Large \boxed{ \boxed{\boldsymbol{Rule : a^{-1}=\frac{1}{a} }}} \\\\\\\\ (2+3)^{-1} \times (2^{-1}+2^{-1}) = \\\\1)\ (2+3)^{-1}=5^{-1}=\frac{1}{5} \\\\2)\ 2^{-1}+2^{-1}=\frac{1}{2} +\frac{1}{2} } =1 \\\\3)\ \frac{1}{5} \cdot 1=\boxed{\frac{1}{5} }[/tex]

Find the Value of y. 70 60 65 40

Answers

Answer:

125

Step-by-step explanation:

360-70-60-65-40

= 125

Answered by GAUTHMATH

sin^6x + cos^6x = 1/4

Answers

Answer:

[tex]\displaystyle x = \frac{\pi}{4} + k\, \pi[/tex] for integer [tex]k[/tex] (including negative numbers.)

Step-by-step explanation:

Pythagorean Identity: [tex]\sin^{2}(x) + \cos^{2}(x) = 1[/tex]. Equivalently, [tex]\cos^{2}(x) = 1 - \sin^{2}(x)[/tex].

Rewrite the original equation and apply this substitution to eliminate [tex]\cos(x)[/tex]:

[tex]\displaystyle \sin^{6}(x) + \cos^{6}(x) = \frac{1}{4}[/tex].

[tex]\displaystyle (\sin^{2}(x))^{3} + (\cos^{2}(x))^{3} = \frac{1}{4}[/tex].

[tex]\displaystyle (\sin^{2}(x))^{3} + (1 - \sin^{2}(x))^{3} = \frac{1}{4}[/tex].

Let [tex]y = \sin(x)[/tex] ([tex]-1 \le y \le 1[/tex].) The original equation is equivalent to the following equation about [tex]y[/tex]:

[tex]\displaystyle y^{6} + (1 - y^{2})^{3} = \frac{1}{4}[/tex].

Expand the cubic binomial in the equation:

[tex]\displaystyle y^{6} + 1 - 3\, y^{2} + 3\, (y^{2})^{2} - (y^{2})^{3} = \frac{1}{4}[/tex].

[tex]\displaystyle y^{6} + 1 - 3\, y^{2} + 3\, y^{4} - y^{6} = \frac{1}{4}[/tex].

Simplify to obtain:

[tex]\displaystyle 1 - 3\, y^{2} + 3\, y^{4} = \frac{1}{4}[/tex].

Rearrange and simplify:

[tex]12\, y^{4} - 12\, y^{2} + 3 = 0[/tex].

[tex]3\, (2\, y^{2} - 1)^{2} = 0[/tex].

[tex]2\, y^{2} - 1 = 0[/tex].

[tex]\displaystyle y^{2} - \frac{1}{2} = 0[/tex].

Solve for [tex]y[/tex]:

Either [tex]\displaystyle y = \frac{1}{\sqrt{2}}[/tex] or [tex]\displaystyle y = -\frac{1}{\sqrt{2}}[/tex].

If [tex]\displaystyle \sin(x) = y = \frac{1}{\sqrt{2}}[/tex], then [tex]\displaystyle x = \frac{\pi}{4} + 2\, k\,\pi[/tex] for all [tex]k\in \mathbb{Z}[/tex].

On the other hand, if [tex]\displaystyle \sin(x) = y = \frac{1}{\sqrt{2}}[/tex], then [tex]\displaystyle x = \frac{3\, \pi}{4} + 2\, k\,\pi = \frac{\pi}{4} + (2\, k + 1) \, \pi[/tex] for all [tex]k\in \mathbb{Z}[/tex].

Combine both situations to obtain:

[tex]\displaystyle x = \frac{\pi}{4} + 2\, k\, \pi[/tex] for all [tex]k \in \mathbb{Z}[/tex].

Which number sentence is not true?
A. |-4.5| = 4.5
B. |0| < |-45|
C. |45| > 0
D. |4.5| > |-45|

Answers

Answer:

D

Step-by-step explanation:

The absolute value of a number is the actual distance of the number from zero. So, it is always a positive number. No negative value.

A) I -4.5I = 4.5    TRUE

B) I 0I  < I -45I   TRUE  

Reason: 0 < 45

C) I 45 I > 0         TRUE

D) I4.5 I > I - 45 I     FALSE

Reason: 4.5is not greater than 45

Answer:

D

Step-by-step explanation:

If 2 < 20x - 13 < 3. what is one possible value for x?

Answers

Answer:

one possible value for x is 31/40

Step-by-step explanation:

Let's isolate the 20x term.  Add 13 to all three terms:

2 + 13 < 20x - 13 + 13 < 3 + 13

and this simplifies to:

15 < 20x < 16

Dividing all three terms by 20, we get:

15/20 < x < 16/20

A fraction halfway between 15/20 and 16/20 is (31/20)/2, so

one possible value for x is 31/40.

Check by substituting this into the original equation and checking whether that equation is now true:

2 <20(31/40) - 13 < 3

or

2 < 31/2 < 16, or 2 < 15.5 < 16 (this is true, so 31/40 is a solution)

Find the approximate side length of a square game board with an area of 145 in 2 Plz help!

Answers

Answer:

Side length ≈ 12.04

Step-by-step explanation:

145 = x²

144 is the closest  square, with the root 12

The square root of 145 is approximately 12.04

If my answer is incorrect, pls correct me!

If you like my answer and explanation, mark me as brainliest!

-Chetan K

Answer:

The approximate side length is 12.0 in

Step-by-step explanation:

The area of a square is given by

A = s^2  where s is the side length

145 = s^2

Taking the square root of each side

sqrt(145) = sqrt(s^2)

12.04159458 = s

The approximate side length is 12.0 in

Hassan drove 78 miles in 1ị hours. If he drove at a constant rate, how far did he
travel in one hour? Enter your answer as a whole number, proper fraction, or mixed
number in simplest form.

Answers

Answer:

1 1/3 hours

Step-by-step explanation:

I took the same test

Find the coordinates of a point that divides the directed line segment PQ in the ratio
5:3.
A) (4,5)
B) (-6, 6)
C) (2, 2)
D) (4,1)

Answers

Answer:

C

Step-by-step explanation:

Let the coordinates be (m, n) then by using the section formula we have

m=(5*8+3*(-8))/(5+3)=2

n=(5*(-1)+3*(7))/(5+3)=2

The point is (2,2)

please help me with that

Answers

Answer:

[tex]\frac{16}{81}[/tex]

Step-by-step explanation:

[tex](\frac{27}{8} )^{-\frac{4}{3} }[/tex]

[tex]=((\frac{3}{2} )^3)^{-4/3}[/tex]

[tex]=(\frac{3}{2} )^{-4}[/tex]

[tex]=(\frac{2}{3} )^{4}[/tex]

[tex]=\frac{16}{81}[/tex]

Answer:

16/81

Step-by-step explanation:

a negative exponent means 1/...

the number in the numerator means "to the power of".

the number in the denominator means take the root of that power.

so, we have to take the third root of the expression, or this then to the power of 4, and finally build 1/... if the whole result.

and the sequence is not making a difference.

the third root of of 27/8 = 3/2

this to the power of 4 = 81/16

this 1/... = 16/81

If the curved surface area of a cylinder with height 15cm is 1320cm², find total surface area​

Answers

Answer:

2552cm^2

Step-by-step explanation:

C.S.A=1320cm^2 ;r=?

h=15cm.

[C.S.A. = 2πrh]

(r=1320×7/660=14cm)

Now,

TSA of cylinder = 2πr (h + r) sq

TSA=2×22/7(15+14)=2552cm^2

Graph the relation shown in the table. Is the relation a function? Why or why not?

Answers

Answer:

what can i help u with

Step-by-step explanation:

No; the relation passes the vertical-line test. Yes; only one range value exists for each domain value

Yes; two domain values exist for range

yes; only one range value exists for each domain.

Given the functions below, find f(x) + g(x)
f(x) = 3x - 1
g(x) = x2 + 4

Answers

Answer:

x^2+3x+3

Step-by-step explanation:

f(x) = 3x - 1

g(x) = x^2 + 4

f(x) + g(x) = 3x-1+ x^2 +4

                 Combine like terms

                   = x^2+3x+3

i am having troubles solving this 4(x+3)=x+42 can i get some help please.

Answers

Answer:

x=10

Step-by-step explanation:

Step 1: Multiply 4 with X and 3. You'll get 4x+12=x+42

Step 2: Keep the variable on one side and the number on the other side. you would subtract X and subtract 12. You'll get 3x=30

Step 3: Divide by the 3 on both sides and you'll get x=10

Given:

[tex] \\ ⇢ \tt \: 4(x + 3) = x + 42 \\ [/tex]

Solution:

[tex] \\ ⇢ \tt \: 4(x + 3) = x + 42 \\ \\ \\ \tt⇢ 4x + 12 = x + 42 \: \: \\ \\ \\ \tt \: ⇢4x - x = 42 - 12 \: \\ \\ \\ \tt \: ⇢3x = 30 \: \: \: \: \: \: \: \: \: \: \: \: \: \: \: \: \: \\ \\ \\ \tt \: ⇢x = \frac{30}{3} \: \: \: \: \: \: \: \: \: \: \: \: \: \: \: \: \: \\ \\ \\ \tt \: \pink{ \pmb{ \mathfrak{⇢x = 10}}} \: \: \: \: \: \: \: \: \: \: \: \: \: \: \: \: \: \\ \\ \\ [/tex]

Verification:

[tex] \\ ⇢ \tt \: 4(10+ 3) = 10+ 42 \\ \\ \\ \tt⇢ 40 + 12 = 10 + 42 \: \: \\ \\ \\ \tt \: ⇢52 = 52 \: \: \: \: \: \: \: \: \: \: \: \: \: \: \: \: \: \: \: \: \\ \\ \\ \: ⇢ \purple{ \pmb{ \bold{L.H.S = R.H.S}}}\\ \\ \\ [/tex]

Hence Verified!

In the following diagram, ABCD is a parallelogram. Is AC the bisector of angle BAD? Show calculations and explain

Answers

AC is not the bisector of angle BAD. AC made both sides a triangle and every triangles angles have to add to 180 degrees. So 115+30= 145 meaning angle DAC is 35 degrees. So AC cuts through BAD but one side is 30 degrees and the other is 35(DCB and DAB are the same angle…they both have to add up to 65 degrees)

Answer:

yes

Step-by-step explanation:

in parallelogram ,<A=<C

<C=<D

then <D=115=<C=115

X+115+30=180....TRIANGLE THEROME

X=35

so that,<A=65

<C=65

Can someone help me on this

Answers

Answer:

The choose (C)

F(x)=x/ (x+1)(x-2)

Simplify the following without a calculator: (5)(6+4)

Answers

Answer:

your answer is 50 I hope it's helps you t

Answer:

50

Step-by-step explanation:

(5)(6+4)

5(6)+5(4)

30+20

50

THANK YOU

Let be the density function for the shelf life of a brand of banana which lasts up to weeks. Time, , is measured in weeks and . Incorrect answer icon Your answer is incorrect. Find the mean shelf life of a banana using . Round your answer to one decimal place. Mean

Answers

The question is incomplete. The complete question is :

Let [tex]p(t) = -0.0375t^2 + 0.225t[/tex]  be the density function for the shelf life of a brand of banana which lasts up to 4 weeks. Time, t, is measured in weeks and [tex]$0 \leq t \leq 4$[/tex]. Incorrect answer icon Your answer is incorrect. Find the mean shelf life of a banana using . Round your answer to one decimal place.

Answer:

2.4

Step-by-step explanation:

 Given :

[tex]p(t) = -0.0375t^2 + 0.225t[/tex]

Mean :

[tex]$=\int_0^4 tp (t) \ dt$[/tex]

[tex]$=\int_0^4 t (0.0375 t^2 + 0.225t) \ dt$[/tex]

[tex]$=-0.0375 \int_0^4 t^3 \ dt + 0.225 \int_0^4 t^2 \ dt$[/tex]

[tex]$=-0.0375 \left[ \frac{t^4}{4} \right]^4_0 + 0.225 \left[ \frac{t^3}{3} \right]^4_0$[/tex]

[tex]$=-0.0375 (64) + 0.225 \left( \frac{64}{3} \right)$[/tex]

[tex]$=-2.5 + 4.8$[/tex]

= 2.4

Therefore, the mean is 2.4

Other Questions
You are working for a company that creates special magnetic environments. Your new supervisor has come from the financial side of the organization rather than the technical side. He has promised a client that the company can provide a device that will create a magnetic field inside a cylindrical chamber that is directed along the cylinder axis at all points in the chamber and increases in the axial direction as the square of the value of y, where y is in the axial direction and y = 0 is at the bottom end of the cylinder. Prepare a calculation to show that the field requested by your supervisor and promised to a client is impossible. The dimensions of a conical funnel are shown below: A conical funnel is shown with the height of the cone as 8 inches and the radius of the base as 3 inches. Nisha closes the nozzle of the funnel and fills it completely with a liquid. She then opens the nozzle. If the liquid drips at the rate of 15 cubic inches per minute, how long will it take for all the liquid to pass through the nozzle? (Use = 3.14.) 3.34 minutes 5.02 minutes 13.39 minutes 15.07 minutes Vasco da Gama fulfilled Portugals goal of a sea route to Asia when he reached * Reread Nick's dialogue with Jordan at the end of the excerpt from The Great Gatsby. Do you think that Nick trusts Jordan's ideas about Gatsby? What makes you think so? Cite specific textual evidence to support your answer. Assume the perpetual inventory system is used. 1) Green Company purchased merchandise inventory that cost $16,100 under terms of 3/10, n/30 and FOB shipping point. 2) Green Company paid freight cost of $610 to have the merchandise delivered. 3) Payment was made to the supplier on the inventory within 10 days. 4) All of the merchandise was sold to customers for $23,700 cash and delivered under terms FOB destination with freight cost amounting to $410. What is the amount of gross margin that results from these transactions Write the net ionic equation for the reaction that occurs when equal volumes of 0.546 M aqueous acetylsalicylic acid (aspirin) and sodium acetate are mixed. It is not necessary to include states such as (aq) or (s). Test for symmetry and then graph the polar equation.r=35sin how would one solve this Suppose the amount of protein is at least 8.6 grams. What is the probability that it is more than 8.7 grams WILL GIVE BRANLIEST AND BE SOOO HAPPY PLEASE HELP!!! 30 POINTS SHARE YOUR SMARTNESS!! Choose an organism. Use credible websites to research the classification of that organism and the characteristics its shares with other organisms you selected and full the table with information you gathered. PLSS HELP FAST!! consider the differences in solids, liquids and gasses, and their surface area. which state of matter do you think would usually have the greatest rate of reaction, and why? During this era, what is the first step of the scientific process? Help!! I think I got these wrong, so please help!! Thank you! Being a celebrity - such as famous film star or sports personality - brings problems as well as benefits. Do you think that being a celebrity brings more benefits or more problems?Give reasons for your answer and include any relevant examples from your own knowledge or experience.You should write at least 250 words. 14. In a statistics class with 15 males and 13 females, five students are selected to put problems on the board. What is the probability that:a. 3 females and 2 males are selected? b.all five students selected are males? c. all five students selected are females? d.at least one male is selected? For any event, P(A) + P(not A) = Electricity is distributed from electrical substations to neighborhoods at 13000 V. This is a 60 Hz oscillating (AC) voltage. Neighborhood transformers, seen on utility poles, step this voltage down to the 120 V that is delivered to your house.Required:a. How many turns does the primary coil on the transformer have if the secondary coil has 130 turns? b. No energy is lost in an ideal transformer, so the output power Pout from the secondary coil equals the input power Pin to the primary coil. Suppose a neighborhood transformer delivers 280 A at 120 V. What is the current in the 1.310^4 V line from the substation? I need help I dont understand at all ? What differences would you expect to see in a population made up of individuals whose chromosomes experience crossing-over frequently compared with a population made up of individuals whose chromosomes do not cross over? Please explain your reasoning.